Difference between revisions of "2022 AMC 8 Problems/Problem 20"

(Solution)
Line 1: Line 1:
==Problem==
+
== Problem ==
  
 
The grid below is to be filled with integers in such a way that the sum of the numbers in each row and the sum of the numbers in each column are the same. Four numbers are missing. The number <math>x</math> in the lower left corner is larger than the other three missing numbers. What is the smallest possible value of <math>x</math>?
 
The grid below is to be filled with integers in such a way that the sum of the numbers in each row and the sum of the numbers in each column are the same. Four numbers are missing. The number <math>x</math> in the lower left corner is larger than the other three missing numbers. What is the smallest possible value of <math>x</math>?
Line 15: Line 15:
 
label((0,2),"$9$");
 
label((0,2),"$9$");
 
label((2,2),"$5$");
 
label((2,2),"$5$");
label((2,0),"${-}1$");
+
label((2,0),"$-1$");
 
label((2,-2),"$8$");
 
label((2,-2),"$8$");
 
label((-2,-2),"$x$");
 
label((-2,-2),"$x$");
 
</asy>
 
</asy>
<math>\textbf{(A) } {-}1 \qquad \textbf{(B) } 5 \qquad \textbf{(C) } 6 \qquad \textbf{(D) } 8 \qquad \textbf{(E) } 9</math>
+
<math>\textbf{(A) } -1 \qquad \textbf{(B) } 5 \qquad \textbf{(C) } 6 \qquad \textbf{(D) } 8 \qquad \textbf{(E) } 9 \qquad</math>
  
==Solution==
+
== Solution ==
IN PROGRESS. DO NOT EDIT. THANKS.
+
The sum of the numbers in each row is <math>12</math>. Consider the second row. In order for the sum of the numbers in this row to equal <math>12</math>, the first two numbers must add up to 13:
 
+
<asy>
~MRENTHUSIASM
+
unitsize(0.5cm);
 +
fill((-3,1)--(1,1)--(1,-1)--(-3,-1)--cycle,lightgray);
 +
draw((3,3)--(-3,3));
 +
draw((3,1)--(-3,1));
 +
draw((3,-3)--(-3,-3));
 +
draw((3,-1)--(-3,-1));
 +
draw((3,3)--(3,-3));
 +
draw((1,3)--(1,-3));
 +
draw((-3,3)--(-3,-3));
 +
draw((-1,3)--(-1,-3));
 +
label((-2,2),"$-2$");
 +
label((0,2),"$9$");
 +
label((2,2),"$5$");
 +
label((2,0),"$-1$");
 +
label((2,-2),"$8$");
 +
label((-2,-2),"$x$");
 +
</asy>
 +
If two numbers add up to <math>13</math>, one of them must be at least <math>7</math> - if both shaded numbers are no more than <math>6</math>, their sum can be at most <math>12</math>. Therefore, for <math>x</math> to be larger than the three missing numbers, <math>x</math> must be at least <math>8</math>. We can construct a working scenario where <math>x=8</math>:
 +
<asy>
 +
unitsize(0.5cm);
 +
draw((3,3)--(-3,3));
 +
draw((3,1)--(-3,1));
 +
draw((3,-3)--(-3,-3));
 +
draw((3,-1)--(-3,-1));
 +
draw((3,3)--(3,-3));
 +
draw((1,3)--(1,-3));
 +
draw((-3,3)--(-3,-3));
 +
draw((-1,3)--(-1,-3));
 +
label((-2,2),"$-2$");
 +
label((0,2),"$9$");
 +
label((2,2),"$5$");
 +
label((2,0),"$-1$");
 +
label((2,-2),"$8$");
 +
label((-2,-2),"$8$");
 +
label((0,-2),"$-4$");
 +
label((-2,0),"$6$");
 +
label((0,0),"$7$");
 +
</asy>
 +
So, our answer is <math>\textbf{(D) } 8</math>.
  
 +
~ ihatemath123
 
==See Also==  
 
==See Also==  
{{AMC8 box|year=2022|num-b=19|num-a=21}}
+
{{AMC8 box|year=2022|num-b=18|num-a=20}}
{{MAA Notice}}
 

Revision as of 19:15, 28 January 2022

Problem

The grid below is to be filled with integers in such a way that the sum of the numbers in each row and the sum of the numbers in each column are the same. Four numbers are missing. The number $x$ in the lower left corner is larger than the other three missing numbers. What is the smallest possible value of $x$? [asy] unitsize(0.5cm); draw((3,3)--(-3,3)); draw((3,1)--(-3,1)); draw((3,-3)--(-3,-3)); draw((3,-1)--(-3,-1)); draw((3,3)--(3,-3)); draw((1,3)--(1,-3)); draw((-3,3)--(-3,-3)); draw((-1,3)--(-1,-3)); label((-2,2),"$-2$"); label((0,2),"$9$"); label((2,2),"$5$"); label((2,0),"$-1$"); label((2,-2),"$8$"); label((-2,-2),"$x$"); [/asy] $\textbf{(A) } -1 \qquad \textbf{(B) } 5 \qquad \textbf{(C) } 6 \qquad \textbf{(D) } 8 \qquad \textbf{(E) } 9 \qquad$

Solution

The sum of the numbers in each row is $12$. Consider the second row. In order for the sum of the numbers in this row to equal $12$, the first two numbers must add up to 13: [asy] unitsize(0.5cm); fill((-3,1)--(1,1)--(1,-1)--(-3,-1)--cycle,lightgray); draw((3,3)--(-3,3)); draw((3,1)--(-3,1)); draw((3,-3)--(-3,-3)); draw((3,-1)--(-3,-1)); draw((3,3)--(3,-3)); draw((1,3)--(1,-3)); draw((-3,3)--(-3,-3)); draw((-1,3)--(-1,-3)); label((-2,2),"$-2$"); label((0,2),"$9$"); label((2,2),"$5$"); label((2,0),"$-1$"); label((2,-2),"$8$"); label((-2,-2),"$x$"); [/asy] If two numbers add up to $13$, one of them must be at least $7$ - if both shaded numbers are no more than $6$, their sum can be at most $12$. Therefore, for $x$ to be larger than the three missing numbers, $x$ must be at least $8$. We can construct a working scenario where $x=8$: [asy] unitsize(0.5cm); draw((3,3)--(-3,3)); draw((3,1)--(-3,1)); draw((3,-3)--(-3,-3)); draw((3,-1)--(-3,-1)); draw((3,3)--(3,-3)); draw((1,3)--(1,-3)); draw((-3,3)--(-3,-3)); draw((-1,3)--(-1,-3)); label((-2,2),"$-2$"); label((0,2),"$9$"); label((2,2),"$5$"); label((2,0),"$-1$"); label((2,-2),"$8$"); label((-2,-2),"$8$"); label((0,-2),"$-4$"); label((-2,0),"$6$"); label((0,0),"$7$"); [/asy] So, our answer is $\textbf{(D) } 8$.

~ ihatemath123

See Also

2022 AMC 8 (ProblemsAnswer KeyResources)
Preceded by
Problem 18
Followed by
Problem 20
1 2 3 4 5 6 7 8 9 10 11 12 13 14 15 16 17 18 19 20 21 22 23 24 25
All AJHSME/AMC 8 Problems and Solutions